What About Your Grades?

Ready to start your journey?

When it comes to college applications, your grades may be the ‘make or break’ factor in getting into many of the nation’s top colleges. But don’t give up hope because  colleges look at more than just your grades.

College Grades

Some questions that you may be wondering include:

  • How important is it to get straight As in your classes?
  • Are AP and Honors classes necessary to look better on an application?
  • What happens when you have a low GPA?

We are often asked these questions (and more) from both high school students getting ready to graduate and adult students looking to head back to school.

Editorial Listing ShortCode:

Regardless of where you are in life regarding college vs. high school, there is no better time than now to start thinking about your college future.

Colleges Look at More than Just Grades

There are many things that colleges look at when considering admission, and when it comes to grades they examine the following factors:

Your grade point average is arguably the most important aspect of your college application, and by understanding how to manage your grades during high school, you can have a better chance of getting into your first choice of college.

GPA

    Students taking exam

    The GPA system can be a bit confusing, and many high schools have different ways of calculating GPAs that make it an even more convoluted process. Some schools operate on the traditional 4.0 system, while others offer “extra” points for more advanced classes (or use a 4.3 GPA scale). So, what does this mean when applying to colleges?

    Editorial Listing ShortCode:

    First of all, it is important to understand what colleges are looking for when reviewing your application. Since it is so common for grade point scales to vary from school to school, most colleges have their own system of weighting GPAs to standardize the admissions process.

    If you have a solid 3.3 GPA or above, then you have a decent chance of acceptance at a wide variety of mid-level colleges, while higher-level colleges often look for a 3.8 GPA or higher.

      If you find yourself struggling in your first year or two of high school but don’t panic. Colleges take into consideration the bigger picture of your GPA and will often focus more on the overall grade trend of your high school career rather than one particular year of study.

      Rigor of Classes

      Arguably, the most important factor when it comes to college admissions is the rigor of classes that you took during high school. The admissions department will take into account your overall GPA when examining transcripts, but they also will look at how hard you pushed yourself throughout your high school career.

      To put it simply, a student with a high GPA that only took the bare minimum of high school credits will not look as favorably in a reputable college as a student with a slightly lower GPA that took almost all AP or honors courses.

      While we don’t advise you to take high-level courses if you don’t feel you will do well in them, it is worth the extra effort to put time into more difficult classes when you have the dedication to achieve an A or B average.

      Standardized Test Scores

        Students preparing for Standardized Testing

        Every high school student is familiar with the standardized testing that has become commonplace for college admissions. So, how much do these scores weigh into your chance of acceptance? The answer depends on the college to which you are applying.

        In a growing trend across the nation, schools are starting to put less weight on SAT or ACT scores or even doing away with them as an admissions requirement altogether. Still, it is a good idea to take these tests during high school to widen your pool of potential colleges.

        Editorial Listing ShortCode:

        Whether you choose to take the ACT or the SAT, colleges understand that these scores reflect your ability to take tests more than your overall intelligence. While standardized testing is important, focusing on a positive high school grade trend carries more weight during the admissions process.

        Explanation of Circumstances

          Some students may be worried about how poor performance during their freshman and sophomore years may affect them when applying to colleges, even if they have turned their GPA around for the better.

          Whether you simply were slacking off during the early years of high school or circumstances beyond your control caused you to struggle with your grades but don’t panic. The majority of college applications leave room for explanations if your grade trends are not what you hoped they would be, and this can be an opportunity for you to give the college a clearer picture of your GPA.

          Editorial Listing ShortCode:

          Additionally, a personal essay, which is often required for admission, is an excellent opportunity to show your future college how you have grown and learned from overcoming obstacles throughout your high school career. Although there are many factors that colleges consider during the admissions process, grades continue to be the primary factor for acceptance into the school of your choice.

          By challenging yourself throughout high school and focusing on maintaining a positive GPA trend, you increase your chances of getting into the school that best suits your long-term goals.

          Frequently Asked Questions

          Students studying in the School library

          I just received my AP exam scores in the mail. I got a 3 in U.S. history but only a 1 in chemistry and physics. I realize that colleges are going to look at all of my scores, but if I did well in the actual class (high As to mid-Bs), will the scores be a deciding factor for admission?

          Would I have been better to have just not taken those two exams, or are colleges going to appreciate the risk, even if the outcome was bad? Also, one specific college I am looking at for early decision only accepts 4’s and 5’s.

          Next year, should I only take the exams that I feel confident I can receive those scores in? I don’t want to take the exam for every AP class I take if it’s actually going to hurt me in the long run.

          Colleges will give greater weight to your actual grades in the classes than to your AP scores, at least for admissions purposes. Course placement is another matter, most colleges won’t give credit for anything below a 3.

          Having said that, I would recommend taking only those AP exams you feel confident about in the future. It looks much better to have fewer, higher AP scores than a bunch of lower scores. In a nutshell, don’t worry about the scores you’ve already received, but take only the exams you feel confident about this coming year.

          I’m a sophomore in a highly-ranked public high school. I have always scored in the top percent of all high school students in the nation on standardized tests (including a composite score of 31 on the ACT in 7th grade).

          students taking notes during class lecture

          I am taking several AP classes (including AP calculus) and am signed up to take 5 more next year. I am also heavily involved in music, ballet, and volunteer work. However, I have been having a lot of personal problems at home for the past year.

          In the second half of my freshman year and the first semester of my sophomore year, my grades dropped from a 3.9 average to a 3.15 average.

          I’m hoping to ace my junior and senior years, as well as my ACT and SAT, but I’m worried about how my freshman and sophomore grades will affect my college admissions chances. I want to go to a top school, though not necessarily an Ivy League. Any suggestions?

          Editorial Listing ShortCode:

          Well, all you can do now is work as hard as you can to improve your grades. If you’re successful in doing so, then you could write a letter to the colleges to explain why your grades went down.

          Assuming it’s a plausible reason, and assuming that you’re able to increase them back up to where they were, you will have done your best to minimize the damage from your grade decline. And, colleges will appreciate your candor. That would be my approach.

          I am a high school sophomore and I’m planning my classes for the last two years of high school, but I’m unsure of what classes to sign up for. Would AP or average-level classes be better?

          Would it look better on my college applications to have possibly lower averages in AP classes or to take regular classes and have possibly higher averages? Which would colleges look more favorably upon?

          Students discussing during their break time

          It’s hard to answer that for sure. It depends on the colleges, your grade point average, SAT scores, etc. The AP courses definitely “look” good.

          Editorial Listing ShortCode:

          However, my opinion is that I wouldn’t take the AP course unless you’re confident that (1) you’ll get at least a B in the course, and (2) you plan on taking some of the AP exams. I’d also suggest taking something in between AP and regular such as honors courses.

          My son has a solid and consistent 3.3 GPA and is in the second semester of his senior year. He has been accepted at Chico and San Jose State Universities, small private colleges in Oregon and Colorado State.

          Despite a lot of help and effort, he got a D- in his first semester in Precalculus. He also got 2 B+’s and 2 C+’s. How much should we worry that these schools will rescind their offers of acceptance?

          It depends on the school. Most make their approvals conditioned on satisfactory completion of the student’s remaining high school courses and receiving their degree. This may involve not getting below a “C” on certain required courses.

          In general, it is very rare for a college to rescind its offer, however, I advise that you check with the school your son wants to apply to and find out the specific requirements after being admitted.

          I believe I heard once that if you receive a certain grade in an AP class college admissions give extra points or boost the score up. If not, how do they treat them?

          Students taking their AP Class

          Typically, a college gives college credit for AP course exams with a score of 3 or better. The exam part of an AP course is optional, but why take the class if you’re not going to take the exam? Each college, though, has its own system for how they treat AP courses. You’ll have to find out from the specific college.

          Currently, I am a senior in high school. I received a 29 on my ACT and have had straight mid-As from my sophomore to senior years. I’m in numerous honors and AP classes. My concern is that in my freshman year of high school, I received a 1.7 GPA. A compelling story, eh?

          I am just wondering how much my awful freshman year will impact my chances of admissions to schools like Columbia, Brown, and other top-notch schools or whether they will notice the dramatic improvement and basically not hold that freshman year against me. Thanks.

          It’ll count, but you know, it’s better to get 1.7 in your freshman year than in your senior year. However, colleges are big into improvement, so I think in some ways, it will help you stand out.

          If there is a compelling story, tell it in your essay. You also need to apply to some safety schools. Hope for the best, but be prepared.

          I am a teacher/counselor at a small public magnet school in Ann Arbor, MI. We have just switched over to a 3-year integrated science curriculum, which is replacing earth science, biology, and chemistry.

          Will this new curriculum hurt our students in admissions? Many fear that if their transcript doesn’t say “chemistry,” they’re doomed.

          Students working on a group project at school

          Well, maybe not doomed. There’s a lesson here for students applying to college, though, which is, “Don’t assume your school’s curriculum is known or understood by each college to which you are applying.”

          Editorial Listing ShortCode:

          When an alternative curriculum like you’ve described is involved, the student’s responsibility is to take charge of the situation. The student should advise the high school guidance counselor that his or her application is going to certain colleges, and the student should question the counselor about whether the various college admissions offices are going to be familiar with the high school’s unique curriculum.

          If not, the student needs to ensure that the school contacts those colleges to update them about the curriculum. Otherwise, getting passed over is certainly possible.

          I’m going to be a junior in high school this year, and I was wondering what colleges look at more often, the level of the classes or your grade point average. I’ve been in honors and G/T classes but recently had to drop to regular English.

          My GPA rose quite a bit, considering I was barely passing honors English, but I got an A in regular English.

          Basically, I think you have to take the best course that you can get the best grade in. There’s no sense in flunking honors English. Colleges do consider the quality of the course you took, and more selective colleges, of course, have higher expectations.

          Editorial Listing ShortCode:

          Many students feel that every course they take has to be an honors class. You might want to cut back on the difficulty of some classes while challenging yourself in others. Some of us are great in English, others in math. Colleges understand that.

          I am a student who has a high SAT score (in the top 1%), but a class rank that is barely in the top 10. I do, however, have other activities to which I am dedicated to such as sports, a job, and a club or two.

          I know that higher-end schools consider a low GPA and a high SAT a waste of potential, but would the top school still accept students like myself?

          two students walking in the campus

          Few students are “perfect” across the board. A high SAT and a ranking in the top 10% might mean you’re simply in school with a lot of smart people.

          It might also mean that you attend a lousy school where achievement at the high end is not very challenging and so students are bunched together at the top. Unfortunately, most students have no idea how their high school stacks up against others.

          You could get some ideas, though, by asking the guidance office where other students have applied and been accepted (like in prior years).

          I go to a private, Catholic high school in Massachusetts. I will be a junior in the fall and will be taking 2 AP classes. I have always done well in my classes and on standardized tests, but I am somewhat anxious as to whether or not I’ll be able to get into the college of my choice.

          I’m not involved in any sports, but am heavily involved in at least 4 academically-oriented extracurricular activities. Will not being on any team hurt my chances of getting in? Will my test scores, grades, and rank be enough?

          Test scores, grades, and rank are not enough. Highly selective colleges are looking for well-rounded students with diverse interests. Even your extracurricular activities sound as if they are academically oriented. On the other hand, that doesn’t mean you need to play a sport.

          Some of us are not sports-minded. Being just a junior, though, gives you two years to develop an interest and pursue it. You may also have great qualifications that you are overlooking, too. Do you play a musical instrument well? Have you traveled extensively? I would suggest meeting with your school’s guidance counselor to explore how you can round out your qualifications without sports.

          The international high school where I attend is considering going to a non-graded system of evaluation. This means there will be no grades on a transcript. Where can I find out about other schools that use this system, and what they do to best inform colleges about their students?

          If your school selects this method of grading, they need to have a plan in place for educating college admissions counselors on how to evaluate prospective applicants from your school.

          In addition, it is my opinion that a gradeless transcript will serve to accentuate other aspects of your application, for better or worse, including things like standardized test scores, class rank, extracurricular activities, essays, and more. You should feel free to discuss your concerns with your high school guidance office. Absolutely do not let them blow this off with, “Oh, colleges know about us.”

          Editorial Listing ShortCode:

          As for other schools using a similar system, I’d ask for a list. Your school must be making decisions on the basis of some information, and you should know what it is. Once you identify other schools, don’t hesitate to call their guidance office and ask what they do to inform colleges about students, and most importantly, to what schools their students are successful in gaining admissions. If it’s a weak list, take note.

          I am a junior in high school. I have maintained a 4.0 and above until this semester. My GPA has dropped to 3.5 because of personal problems with studying.

          High school students during an exam

          I feel that I can substantially raise this average this semester but I am concerned that without some explanation to admissions my chances have lessened in applying to specific colleges of my choice. Do you feel that this effort is worthwhile?

          Sometimes, explanations come across the wrong way. As a junior, you still have time to improve your GPA, although a 3.5 is nothing to sneeze at. You might be able to make colleges aware of your personal challenges without sounding like you’re crying foul.

          Editorial Listing ShortCode:

          If you’ve overcome some obstacle in your life, perhaps your essay is a place to describe the challenges you’ve faced and how you dealt with them. I think that is a much more powerful approach than attaching a note to your application saying, “I would have done better if only…”

          I am a sophomore in high school and am trying to decide whether I need to take my third year of Spanish. Are 2 years of a foreign language enough to be competitive with most liberal arts colleges? If it matters, I have a 3.0 and 1120 on the PSAT.

          Language requirements vary from one school to the next, and often from one program to the next. The only way you’re going to know for sure is to investigate specific opportunities.

          As an aside, as we become more of a multi-cultural society, the ability to converse in multiple languages is a plus. It will help you, perhaps, more in your career than in your college education. If Spanish is something that you’re doing well in, you might consider another year for reasons beyond college admissions.

          I’m a senior, and I’ll be applying to very competitive schools this year and all that jazz. My grades have always been pretty good, like in the 90s, except for my math, which has always been in the 80s or 70s.

          Three students talking in the hallway

          I’m just really bad at math, and consequently, it has lowered my average every time. Will colleges look at my average and say, “We’ll pass on this guy,” or will they say “He’s just bad at math”?

          Colleges will look at your overall GPA much more so than individual coursework averages. The exception, of course, would be that if you were applying to a math-related field, for example, your math grades would be examined more closely.

          I don’t think you’ve done yourself any favors in applying to “very competitive schools” with math scores in the 80s and 70s. These schools are simply able to select students who have all the tickets, and unfortunately for you, there are plenty of them to choose from. However, I would encourage you to apply to your top two choices anyway.

          Beyond that, find schools that specialize in the academic areas in which you excel. Straight As in math this year would help a lot, too. Good luck.

          Are mid-year reports considered by colleges? I have some mid-year Cs.

          Students attending class and reading notes

          Not likely. But it depends on how your particular school cumulates grade point averages. If like most schools, it is at the completion of a course and based on your final grade, then you’re OK.

          Editorial Listing ShortCode:

          However, if they are averaging semester grades, then you have a different situation. But this is a factor in your overall GPA. I can’t imagine a high school releasing mid-year grades on specific courses. Go to your school guidance office for a better understanding of how your school operates.

          Our school district is currently studying AP and weighted grades. As a parent on the committee, I am trying to determine the importance of both of these issues as they relate to our students’ ability to compete with others to get into college.

          Students in group study at the library

          One of our high school counselors stated that weighted grades were not as important as class rank. Is this true? How important are AP classes? Don’t most schools weigh AP classes? Any help with these issues would be greatly appreciated.

          These are heavy-duty issues largely beyond the scope of this site. However, some trends might be of interest to you.

          First, while the grade a student gets in his or her AP class might be weighted, the results of testing make that weighting almost irrelevant. These tests are administered by The College Board, are standardized, and the scores can be compared without prejudice.

          As for class rank, many high schools are dropping them. They are helpful to a college, but high schools find that colleges sometimes have a mental cut-off, that is, for example, not taking students below 10th, and thus, a good student may not get in because of his or her ranking. I personally would like to see class ranking dropped.

          As for grades, sadly, colleges are relying more heavily on standardized tests to determine a student’s ability to do the work at their school. Because grade creep has made average students seem better, grade point averages often don’t say much about the student.

          Editorial Listing ShortCode:

          One admissions counselor recently commented, “Grades tell you zip.” That’s too bad. Even though I’m not a fan of the SAT or ACT, their importance seems to be growing. Weighting your grades may make your students look better on paper, but it may not make them more competitive. College admissions counselors know the ropes, and I think they see through the smoke and mirrors.

          I was wondering if you could please explain the difference between a weighted and unweighted GPA. What do high schools do to your GPA to weigh it? Which is more important to the colleges:  your unweighted GPA or your weighted GPA? Thanks.

          Attentive students during class lecture

          An unweighted GPA means that a school gives you 4 points for any class in which you make an A (3 for a B, 2 for a C, 1 for a D) regardless of whether it’s an honors AP class, etc. A weighted GPA means that your school gives extra points for certain classes (usually advanced, honors/AP/IB classes).

          With regard to your last question, colleges will recalculate your GPA so as to level the field between students whose schools calculate GPAs differently. So you don’t need to worry about which one they pay more attention to. Just worry about earning the best grades possible. Good luck!

          Do colleges look at each of your semester GPA, your year’s GPA, or your overall GPA consisting of your freshman, sophomore, junior and senior year? Thanks!

          Students working together for their group project

          Admissions committees will consider each of these things, but they are usually most interested in your overall grade trends. This means they’re not just interested in your overall GPA but in how you perform from one semester to the next and one year to the next in both individual courses and overall.

          That said, it’s important to know that admissions committees typically recalculate your GPA using a formula they come up with. This way they can account for the different ways schools calculate GPAs and enable themselves to compare different students. Good luck!

          I know this is a very broad question, but in general, what do colleges look for? Do they look for your GPA and your ACT/SAT test scores? And do they look at your school exam scores at all?

          Admissions committees evaluate applicants primarily using six factors: grades, the rigor of curriculum, essays, recommendations, extracurriculars (and leadership in those activities in particular), and SAT/ACT scores.

          Editorial Listing ShortCode:

          So yes, they do consider your GPA and SAT/ACT scores, but these are by no means the only or even the most important factors. Each of these six factors is very important, though it’s impossible to say what percentage each counts. To the best of my knowledge, they will not look at your school exam scores. Good luck!

          I’m a freshman in high school. Do colleges and universities look at your exam grades or just your general grades? Thank you very much.

          Typically, admissions committees only consider your overall grades and grade trends. (I don’t even know if your exam grades go on your transcript.)

          It’s also worth noting that applications include a space for you to explain any unusual grade trends. If you’re concerned about your exams lowering your semester averages, you may want to use this space to explain these grades.

          Just make sure you also show admissions committees how you’ve overcome this problem and assure them that similar problems won’t affect your college performance. Finally, keep in mind that you’re only a freshman. While your grades are important, admissions committees pay more attention to the grades you earn in the latter half of high school than they do to those you earn during your first year or two.

          They know many students struggle to adapt to high school life and that junior- and senior-year courses are more comparable to college-level courses. Good luck!

          I am currently paying off tuition from last semester, but I do not like my school or my grades. So, I am thinking of stepping down and going to a community college.

          I do not want to tell them that I attended college before. Is this a crime? Also, at the university that I was in and am paying off, I got a grant via FASFA.

          Students talking in the hallway of a community college

          I want to apply to the community college as a first-time student and do another FASFA. Would I get in trouble for lying that I have not attended any college?

          Yes, you would get in trouble. Don’t think about doing it. The new college might be able to track your past attendance down in a second since you received a grant anyway. Besides, most colleges ask every student to list any other past institutions they have attended. You do not want to start off your new college career by lying.

          Editorial Listing ShortCode:

          Your best bet is to explain any extenuating circumstances to the admissions office or in your application and present reasons for your fresh determination to succeed. (Since you have to fill out the FAFSA every year, you may still receive financial aid for this next year.)

          Again, honesty equals integrity. If you want to have the hope of being a person of good character, I urge you to remember this. By the way, if you are attending a community college, you may have chances to highly improve your grades and replace your old grades for core courses with new ones. So make sure to check into that option as well.

          Colleges aren’t against you, so you don’t have to lie to them and treat them as the enemy. Your college wants to help you succeed and will provide you with the help you need if you do your part to seek that help out.

          I’m a freshman in college and just got back my midterm grades. I’m a pre-med major and want to get into a good medical school, but my grades have been kind of bad. I have one F and a couple of Ds on there, but I have been doing better on later tests.

          freshman student studying at school library

          I was wondering if there was time to redeem myself or if I am too late. Also, if you could give me advice on college life in general and study tips, that would be great. Thank you in advance for your help!

          Well, I wouldn’t say it’s ever too late! But you’re right to start busting your butt now. The best thing you could do to get help in college life and studying is to look over our whole website.

          There are lots of resources we’ve written up in the past on MyCollegeGuide.org that will advise you on college life and study principles. But honestly, one of the best things you can do to improve your study skills, study habits, and your own schedule boundaries is to get somebody who will hold you accountable to sit down and do your work.

          Editorial Listing ShortCode:

          Think of it as finding a personal trainer for academics instead of athletics. If you have somebody who will push you to stay focused and complete your work, who will motivate you from the outside, that’s going to be your best bet. Whether it’s your roommate, an adviser, a mentor, or a trusted adult in town, get somebody who won’t be afraid to push you and keep you on track.

          My son (who is a rising senior) has a 4.2 weighted GPA, 1950 on his SATs, and will have taken ten AP classes and three honors classes by the end of his senior year.

          My question: Will a C+ on my son’s transcript disqualify him for admission to prestigious schools? He took AP Calculus as a junior and got a C+ the whole way through.

          Teacher showing his students the result of test

          Someone mentioned to me that some schools will simply see the C on his transcript and reject him regardless of his GPA. Is this true?

          It’s hard to believe that given his achievements that a C+ (which by the way is an 83% in his school) will keep him from admission to a top-tier school (not necessarily an Ivy).

          He’s a member of the National Honor Society and Spanish Honor Society, has won numerous academic awards, and is a 4-year varsity pitcher who is gaining some recruiting interest. What do you think?

          It does seem a bit extreme, doesn’t it? I wouldn’t necessarily say that your son is automatically disqualified from all top-tier schools because of this one grade.

          It sounds like he has great standing overall in the various areas of his high-school career. But it is true that some schools go through so many applications that they have to use at least something as an automatic cutoff, and in a few cases perhaps it could be a grade. But I really wouldn’t worry about it since his GPA is otherwise excellent.

          Students doing research works together

          If the grade is still a concern, then your son should feel free to appeal to his teacher for extra work or an alternative way of raising his grade for this particular course.

          He may also want to consider attempting to raise his score on the SAT if he feels it is a concern. I urge you both to also remember that attending one of the top colleges is not a necessity to succeeding in life. There are plenty of wonderful schools in the country, and some you might not have thought of could be an ideal fit for your son (especially considering his baseball talent). My best to your son and to your family.

          It’s my senior year in high school, but I have a D in chemistry for the 1st semester. Is there any possible way I can still go to a 4-year university?

          Of course. You don’t need a straight-A transcript in order to attend a four-year college! That’s absurd. Many students think that one low grade will condemn them to community college forever. It’s simply not true. That being said, of course, work to improve your grade. Might as well do the best you can.

          I am a junior in high school, and I have done horribly all 3 years! I mean like 1.9! I’m tired of goofing off and want to get my head on straight. If I do well in my senior year, will a university accept me?

          Some students studying together

          You still have some hope. There are state universities and other four-year universities that are not as selective as some other colleges, so ask around to find out which colleges those are. And yes, definitely do as well as you can with the rest of your junior year and your whole senior year.

          Prepare well for your SAT/ACT test(s) and do the very best you can. Talk to your teachers and see how you can improve your current grades (extra credit? staying after school to get some extra help?) Talk to your guidance counselor.

          Editorial Listing ShortCode:

          Take the initiative. Your teachers want to help you succeed. Use them as a resource. Get your parent(s) or another trusted adult to hold you accountable to complete your assignments. Working hard feels really good once you’ve done some of it. It’s even kind of a rush.

          Most of all, don’t give up hope. Lots of teenagers who are struggling go on to do very well in college. It sounds like you have developed the mindset to turn things around and do the same. But you must take action. Don’t just think about doing well and then slip back into laziness. You have some control and some choices, rise to the challenge! Now is the time to become the adult you could be! My best to you.

          I am a junior. If your sophomore grade in English is better than your honors English, which teacher is better to ask for a recommendation?

          Group of students reviewing their English lecture

          Thanks for your question. If the two grades are somewhat comparable and one is a little better than the other, I don’t think you need to base your decision on that. Instead, which teacher did you connect with more? Which one might be able to talk about your skills and strengths a bit better?

          I would base your recommendation decision on those factors rather than simply which gave you a better grade. So, if the honors English teacher knew you better, then don’t be afraid to ask that teacher. Good luck.

          Hello! I’m currently in 11th grade in high school, and I need help deciding whether I should take choir or psychology next year. I don’t have a lot of extracurriculars, so I wanted to join the choir to have a class to enjoy rather than doing bookwork all the time.

          However, taking choir would bring down my GPA, which is currently 4.6/5.0 (though I’m not even in the top 10% of my class). If I take psychology, it will bring my GPA up about 0.1 but I will have one less extracurricular.

          Is it better to be more involved and take choir (does it even account for much) or aim for a better class percentage by taking psychology?

          Here’s my honest advice: At this point, especially since you have above a 4.0, I think you should take the class you want to take more. Take the one that will give you energy and joy, whether that’s belting out classical tunes or unraveling the mysteries of the brain.

          If you learn how to be true to what you truly enjoy instead of trying to please colleges, that will probably take you farther than your GPA ever could.

          Is a WP on an HS transcript in freshman year a really big red flag that will deny admission to all good colleges even if the rest of the transcript is good?

          I am thinking of dropping down from an honors course to regular to improve my grades, but it will give WP notation on my transcript. Should I stay in Hons and keep struggling, or risk a WP? Please help. Thank you.

          Students working in a group study

          You’re right that it’s typically better to work at keeping up your grades in an honors or AP course rather than dropping down. But of course, if you’re getting an F, that might be a different matter.

          I would talk to your honors teacher first before anything and see if you can get some extra help outside of class or get some kind of extra chance to raise your grade in any way. That might include asking a parent or trusted mentor to help you study. But typically teachers are happy to help you work hard and improve your grades.

          They want you to succeed, so take advantage of that possibility first before you would drop down. Good luck.

          I didn’t do well in high school. I attended community college for two semesters and still couldn’t seem to pass the remedial courses. I’m a long shot from a dummy, but as of right now I don’t know what would be my best options for going to a university. Help me!

          This is only an idea since I don’t have too much information from your email, but it could be that you just have to adapt your study habits, or perhaps you have a different learning style from most people.

          Do you have trouble concentrating in class? Are you antsy and can’t sit still? Are there ways you’ve noticed you do focus better (for example, after taking a nap or going for a run – or maybe you need the room to be completely silent)? Do you have to schedule your study time for a specific time of the day?

          Editorial Listing ShortCode:

          Maybe you learn better by listening, or maybe you have to be looking at a picture or diagram. I would reflect on how you concentrate and learn best because it could be that you’re simply a smart person who doesn’t have the same learning style or the same needs as a traditional classroom learner.

          Ask people around you to give you input as well. Friends and family can usually offer good insights. Also, many universities offer specialized study and note-taking options for people who just need a different kind of support.

          It could be that once you figure yourself out a little more you could go on to a university and do very well. Or you might find that you like the idea of attending a vocational school even more. I hope this at least gives you something to chew on.

          Hi, I am currently a junior at a very good high school. My past two years have been dreadful, and I failed nearly all my classes, but I am doing better already and wondered if there’s any hope for me to get into a university because my dream is to be a nurse. But is it already too late for me?

          Students taking notes

          It’s certainly not too late for you. Definitely, keep your grades up as high as you can, and if you are able to go back and retake some classes during your first two years (in summer school or an option like that), go ahead and do that too.

          Something else I would suggest is that you start volunteering at a hospital or in hospice care to assist people and take care of them in a medical setting. If you don’t have time to do that during the year, do some volunteering in the summer. That kind of experience would show colleges and nursing programs that you are serious about wanting to be a nurse.

          Once you graduate high school, certainly apply to some four-year universities. You can apply to some community colleges too if need be, but you might not have to worry about it. Either way, the most important thing is that you show a steady track of academic improvement/high grades during the next few years. That, too, will show your commitment to being a nurse.

          You’re still young, and you still have time to turn things around. Go for it!

          I am a high-performing student. I take the full IB Diploma track and I’m involved in several extracurriculars/groups. I have risen to leadership in at least three of them. However, as a junior, my grades are going from As to Bs, and I have stepped down from a leadership position.

          students studying in the library

          To be frank, I do not have a strenuous situation or crisis, and I have just been struggling with the motivation and focus to do well.

          Besides improving my GPA, what should I do to better my chances of getting accepted into a top state school? I am afraid that college admissions will see the decline in my transcript and mark me as a slacker or a burnout.

          Ah, yes. School starts to get a little autopilot-feeling around this time, doesn’t it? That sounds totally normal to me.

          Well, it sounds like you’re already taking the necessary steps to avoid becoming a burnout at age 17. I don’t think it’s a deal-breaker to step down from a leadership position, especially if it will help you keep your grades up. At this point (since you already have leadership positions in some other things), I think it’s a good idea to focus on maintaining your GPA.

          Editorial Listing ShortCode:

          To focus on motivation,  have you envisioned one or two schools that you’re really interested in? Have you been able to imagine yourself walking through the gates of that campus? That might help as a one-second motivational tool whenever you’re feeling kind of blah about life or school. Also, have you ever kept a journal?

          You might want to start tracking or reflecting on whenever you feel down,  you may find that you’re able to trace it back to a certain time of the day (or time of the year), or that it’s tied to interactions with certain people, or that there are certain pathways that lead to slipping into weird thought patterns.

          I really think that keeping a journal for a while could help you process all the different things swirling around in your brain. But, chances are you’ll probably also need somebody to help you stay up with things, not just your own imaginations and reflections.

          Do you have a friend who will work hard with you but also take a break and enjoy a Friday night movie with you? Can you ask a parent or sibling to check in on you and ask how you’re feeling and help you stay disciplined?

          Students answering their assignment together

          I think gathering a support system, at this point, might be the single most helpful thing you can do for yourself at this moment. And, don’t worry, working through a lack of motivation and challenging yourself to stay strong is an essential part of becoming an adult. Sounds to me like you’re on your way there.

          Is it better to go to a competitive high school and get mediocre grades or a less competitive school and get good grades? Do you think class rank matters?

          It is best to attend a high school that will challenge you, and one that matches your level of academic ability. Class rank does matter, but colleges take into account the rigor of the curriculum at each high school.

          Editorial Listing ShortCode:

          Therefore, it wouldn’t help to attend a school that is less competitive in order to rank higher. You should attend whichever school is a good fit for your abilities, talents, and interests. Work as hard as you can to excel and keep your grades high. This will show your firm commitment to your education. Good luck!

          I am a sophomore in high school. During my freshman year, I earned mostly As and held a 3.9 GPA. I was accepted into my school’s special program, but it was a lot more intense than I am used to.

          Students reading a book on the campus ground

          My grades dropped dramatically. I now have around a 2.5 GPA, and I am worried about whether or not prestigious colleges will still accept me. If I get straight As during my remaining years in high school, how will colleges use that in considering me?

          This is a great question. First, it’s admirable that you started strong and entered an accelerated program. Unfortunately, you don’t always know how rigorous a program will be until you try it, and it sounds like this particular program was not a good match for you during your second year.

          The first step is to adjust your schedule and take classes that match your academic level. Since you know that this special program is a bit too intense for you, dial back to classes in which you know you can earn a B or higher. The idea is to take classes that challenge you to grow, but in which you can still earn high grades. As for your questions about colleges, any school will look at your application as a whole.

          If you pull your grades back up to a high level, you will demonstrate that you are a hardworking student who simply took on too much responsibility in sophomore year. Focus on improvement all around, and make sure to balance your studies with extracurricular activities. Good luck, and stay focused on improvement!

          students researching at school library

          I am terrified of college acceptance. I fear that I will not be accepted into any school. My overall cumulative is 3.0, but I had two Ds in my junior year. I’m in the science and technology program at my school, so the work is extremely challenging.

          I have 137 community service hours, 6 clubs, and 1510 on my SAT. I am now a senior in high school. What are my chances of going to prominent schools?

          It sounds like you experienced a decline in grades junior year, but other aspects of your record are quite strong. Don’t be discouraged, and don’t give up. However, do sit down and assess the reason why your grades fell and be prepared to talk about strategies for improvement and lessons learned when you write your application letter.

          Editorial Listing ShortCode:

          I can’t comment on your chances of getting into any school, because it’s just not possible to quantify chances. However, I can advise you to cast a wide net and apply to prominent schools, high-quality state universities, and an array of programs that match your interest. Only aiming for Ivy Leagues or top-tier schools can really limit your chances since competition is tough at these schools.

          Make sure you leave yourself options and continue improving and working hard throughout your senior year to show your commitment to your education. Good luck!

          I am a junior in high school with a 4.5 GPA. I am in two AP classes and an honors language class this year.

          I play volleyball at my school and am involved in elite student organizations. However, I am unsure about my schedule for next year. I am set on taking three AP classes, but I am not sure if

          I am going to take a science. The class would be honors so it will look good, but if I choose not to take a science my senior year and take a free period will it hurt my chances at highly accredited schools?

          It sounds like you have a great academic record with strong grades. If you’d like to take a free period instead of another science, consider how you could use that time toward something constructive. For example, if you are using the free period to pursue an extracurricular activity, that’s a positive thing that could strengthen your application.

          Editorial Listing ShortCode:

          I would discourage you to take a free period just to hang out, though, as this might signal that you’re not ready to manage your time in college and use your hours toward productive activities. You don’t necessarily need to pack another science class into your schedule, but do use that class period for studying, preparing for college, or working in a club or organization. Good luck!

          I am a parent of a junior. She currently has a weighted GPA of 3.78 and has taken all honors courses available for all the core subjects offered.

          She recently took the SAT, and she is planning to study more over the summer to retake in the fall, hoping to bring up to at least 1100. She is planning to take dual enrollment college English next year. She has never taken an AP course.

          students checking options on dual enrollment

          We are debating if she should take an AP class next year. Will taking one AP course make a difference? She is pretty well rounded in the extra-curricular activities and volunteering and she has a few academic honors.

          That’s a great question. Typically, it’s best to only take AP classes if a student can achieve a ‘B’ or higher. If your daughter is challenged adequately by honors classes, then that’s a good indication that she’s studying at the appropriate level.

          However, if she finds honors courses easy to manage and feels ready to take on a course that more closely matches the rigor and workload of a college-level class, an AP class is a good idea. It’s a very important decision, though, because a low grade in an AP course can lower the GPA and signal to an admissions committee that she might not be ready for college-level work.

          Editorial Listing ShortCode:

          Rather than focusing on how it will help boost her chances, focus on whether she’s ready to take her academic work to the next level. If not, there’s nothing wrong with graduating with honors courses, a high GPA, and plenty of extracurriculars.

          My advice is to focus heavily on grades and the SAT. A higher SAT score will certainly help, and if sticking with honors courses will allow your daughter more time to raise that score, I’d suggest focusing on the test preparation. Not having an AP class or two on the transcript won’t lessen her chances, but they do supply great preparation for college. Good luck!

          I have been struggling with trigonometry all year. I’m just not that great at it. I think I’ve only been getting a 70 average in this subject. Do colleges ever judge students only on math scores? What about during junior year?

          Students taking notes in class

          I’m sorry to hear that you’re struggling with this math course. But understand that no college will look solely at your math scores to measure your competence and potential as a college student. It’s more important for you to focus on solutions to the problem.

          If you’re doing everything you can to keep your grades high, a college will understand that you are a hardworking student with different strengths and weaknesses. Are you working with a tutor or soliciting help from your teacher? Also, if you are taking advanced math classes, make sure you’re enrolling in courses in which you can achieve a B or higher. Stay positive, and work as hard as you can.

          No college will rule you out from admissions because of one course’s score. Keep other areas of your academic profile strong, and stay focused on your goals. Good luck!

          I was suspended in ninth grade for a year. I had a pocket knife, but that’s it. There was no violence, no fight, and no incident. After that, I was homeschooled, graduated from high school at 16, became a volunteer EMT putting in well over 1000 hours a year into my squad, became a CPR instructor, and won numerous awards.

          Students reading a book together

          I also enrolled in my local community college where after 36 credits I have a 3.75 GPA. I’m a member of Phi Theta Kappa. Do you think that the one black mark on my high school record will blacklist me from colleges?

          From what you’ve described, it certainly sounds like the incident in 9th grade was not an indication of your overall character. You’ve listed so many great achievements and it seems like you’ve taken your education and public service very seriously.

          Editorial Listing ShortCode:

          Congratulations on bouncing back from what happened in high school! Do not be discouraged, but do go into the application process understanding that your suspension won’t go unnoticed by the admissions committees. Anticipate their questions by discussing the incident in your personal essay. When you talk about it, talk about it in a positive way. In other words, don’t focus on how something happened “to” you that was unfair.

          Focus on what you learned from the event. The school didn’t suspend you because they were out to get you. They likely suspended you because their first priority is to keep all students safe, and bringing any sort of weapon to school is illegal.

          If you can show that you are mature enough to understand, now, what happened and why you’ll mitigate any worry about your character as you enter college. I think you have a strong record and a great chance of attending college. Stay focused on the goal!

          I bombed ninth and tenth grade, literally barely passing anything. I am trying to do better this junior year, but still, things are shakey. I got a good score on my ACT but my GPA is really low. I attend an online school.

          My counselor doesn’t think I will graduate before the end of 2016. Is there any hope that anything besides a community college would want to accept me?

          Three students headed to their classroom

          Rather than worrying about whether a school will want to accept you, consider why you’ve bombed those years of courses, and try everything you can to get back on your feet. Without the proper skills to manage time and work, you’ll likely have a negative experience in college.

          Starting at a community college is not a bad thing. It may help you build your GPA and prepare you for more rigorous coursework at a university. After a year or two, when you’re ready, you’ll have a far better chance of transferring in.

          Perhaps, first, sit down and assess where you are with your career goals and decide what kind of degree you need. Then, determine what factors have gotten in the way of your education. Only when you’re ready to take college courses and treat them like a full-time job should you embark on this journey.

          Editorial Listing ShortCode:

          Good luck! Solicit help and let your friends, teachers, and family know that you need support and want to turn things around. You may be surprised at how much you can achieve with a fierce determination to change.

          I am a senior in high school. I have final exams now. What if I don’t do very well on one or two exams? What could happen when the college sees a low final grade?

          Students walking out of school after class

          Your grades do matter all the way to the end of your senior year. If you’ve already been accepted, it’s important to know that the college can revoke the acceptance during the summer before freshman year. This has happened to students who received failing grades, a very low drop in grades, or an academic irregularity or suspension.

          Don’t let fear hold you back, though. Utilize whatever resources you need to make sure you can ace those exams. Meet with your teachers and explain your concerns. If you work as hard as you can, it’s unlikely that you’ll lose acceptance to a school. Good luck!

          My biggest dream is to be an artist for Marvel Comics. I’m just finishing my sophomore year of high school, and let’s just say my grades haven’t been the best.

          I maintained straight As right up until the last half of the second semester of freshman year when I fell into a state of apathy. I learned later that it was a mild depression brought on by major identity issues that haven’t quite left me yet.

          Now I have to take summer classes to make up the ones I’ve failed (I failed a semester of English, despite it being my absolute best subject besides art) and I’m hovering around a ‘C’ average.

          My pre-SAT scores were within the 95th percentile, and I know I’m going to do well in my junior year because I’ve dropped most of my honors classes except AP Art.

          Students taking their summer class

          However, both my counselors and my parents tell me that because I did so badly for the last year and a half, I’ve ruined my chances of getting into a decent 4-year college and that I should just lower my expectations and go to the local community college.

          Even if I was fine with going to a community college, I don’t want to stay here because one of Marvel’s HQs is in LA, which is 6 hours south.

          To get an internship at Marvel, too, I need to be enrolled in a 4-year college, not to mention that I would feel like a failure if I didn’t get into a halfway decent school. I’m confident in my art and language abilities, but math is one of my big failings because I don’t like it.

          I’m scared for my future and I don’t know how to dig myself out of this hole I’ve made. Will I ever be able to get into a 4-year college if I’ve bombed a year and a half of high school? Thank you for your help.

          Here’s the deal: It sounds like you’ve followed your passions and channeled your energy toward the subjects you love while spending less time working on fundamentals in subjects you don’t particularly enjoy.

          This is understandable, but it’s also a good life lesson. Even when you become a successful comic book illustrator, there will be parts of the job that require math, logic, and tasks you don’t particularly enjoy. However, in order to be successful, you’ve got to be well-rounded and also understand the principles of the work of others who you collaborate with.

          For example, in order to be a good illustrator, you’ll need to know the mathematics and specifications that book designers and web designers require. To excel in graphic design, you’ll have to learn computer software. Becoming a comic artist at Marvel won’t just be limited to drawing with paper and pen. So, while these other subjects may not be your favorite, they are crucial.

          Editorial Listing ShortCode:

          It sounds like Marvel wants an intern who is serious about education and diligent about the tasks at hand. You’ve got two more years to pull up your grades, get serious, and show that you are that student.

          I think you should gear up, put all your energy toward your goal, and learn a lesson from ignoring those other subjects. If you can demonstrate that you overcame your challenges and really want this future career, you can earn acceptance to a four-year college. Nothing is completely certain, but it’s certain that you won’t have a chance if you don’t try.

          You have something that puts you at an edge over other students: a specific goal. Since you know exactly where you want your education to lead you, you have a very specific plan and dream. You can make it happen if you put the work in. If you’re serious about it, tell your family and friends that you need support and encouragement.

          Strengthen all of your grades, nail those test scores, and work on building relationships with teachers who will help motivate you and push you toward your goal. I’d even encourage you to reach out to teachers in those subjects you dislike and inquire about how those skills will help you become a successful comic artist. You may be surprised at what you learn. Good luck to you!

          You probably get this kind of question all the time, but I’ve had a hard time finding the answer to my specific issue. I had trouble in my first three years of high school. I was diagnosed with depression and failed some core classes.

          I scraped by with a D, which is passing at my school and earned me the credits.

          Two students studying on campus grounds

          However, I got an F in an algebra course. I’m a senior in high school and there is no way to make up these classes. I haven’t taken the SAT or ACT yet, but I will. I think I’ll do great on the English portions, but on the math, there may be problems.

          Are there 4-year colleges that will even consider my application?

          I’m sorry to hear that you struggled during the first few years of high school. It may be helpful to consider starting at a community college. This would allow you to build a GPA and record to transfer, later, into a university if you want a four-year degree.

          I don’t want to discourage you, but with those grades, you will need very high test scores and impressive extracurricular activities to be considered by a college or university. Competition is tough, now, and is only getting tougher. Look at community colleges in your area and see if you can find the minimum requirements. Also, study for those tests and consider tutoring for the math sections if you know you need extra help.

          The goal is not out of reach, but it will take some effort for sure! While you may have the ambition and drive to improve, now, you need something that reflects that on your application. Community college might just be the way to show it. Good luck to you!

          My son has an overall GPA of 3.69/4.4 in junior year, but his science (math, biology, chemistry, physics, and statistics) GPA is very high (3.9/4.4). He is going to apply for a science or engineering major. What top schools don’t care much about non-sciences coursework?

          Students are discussing while waiting for the class to start

          A transcript that shows strengths and weaknesses is common, and an admissions committee at any school will be able to decipher whether the student is a strong candidate by using a variety of measurements including test scores, GPA, extracurricular activities, essays, recommendations, and possibly even an interview.

          To answer your question, there is no guarantee that any top school will disregard non-science coursework. In fact, many science and medical programs more frequently consider students with backgrounds in humanities and social sciences.

          My suggestion is to encourage your son to think hard about what field he would like to study and why. Strong test scores and a high GPA leaning toward the sciences will indicate strengths in his chosen field, and a polished essay that speaks objectively about future goals will strengthen the application even further as well as meaningful involvement in a few activities.

          A perfect GPA says very little without these other elements, so don’t worry too much about the imbalance. Your son should pick schools that are a good fit financially and academically, and not worry too much about his GPA. Best of luck to you!

          I am a 22-year-old female who has been working ever since I graduated high school. I was never great at school, but now I want something better for myself. I thought of joining the military and wanted to enter the coast guard but I didn’t take high school seriously.

          Will it be out of the question for me because of my grades? I am willing to learn and turn my life around if given the chance.

          student enrolled in the military

          It is never too late to set a goal, make a plan, and execute the plan. If your goal is to better yourself and find a career in the military, you can certainly reach it. You’ll just have to put in some research and work. The fact that you’ve been working says a lot about your drive and ambition.

          Many students do poorly in some aspects of high school because they are unable to focus on subjects that don’t speak to their passions. If that sounds like you, my best advice would be to isolate your passions. What are you willing to work hard for? And, what interests you enough to keep working, year after year? Remember that a military career is a commitment and isn’t something you can back out of.

          Editorial Listing ShortCode:

          When you’re ready and have done some research and introspection about your interests, find a local recruiting office and speak to someone specifically about opportunities in your area. Good luck, and never give up on bettering yourself!

          Do colleges really care about your high school first semester freshman year grades? I’m not doing good this semester but if I do good for the rest of the years do they still count that?

          Every semester matters and you should do what you can to try to raise your grades now rather than waiting until later. But, the later semesters will be more important to the college admissions committee. Sophomore and junior year grades tell the committee how well you’ll perform academically once you are adjusted to school and in a routine.

          Junior year is also one in which you should be challenging yourself to take courses at the highest level you can while still earning As and Bs. So, while this year is important, it’s okay if you didn’t perform as well as you wanted to perform. But get started on a new path right away by building better study habits and time management skills. You’ll be glad you developed these skills early in the game. Good luck!

          During my freshman and sophomore years of high school, I was unfocused and didn’t care about school as much as I should have. My GPA for both years was under 3.0 and I would like to make up for those years and get into my dream college.

          I am now a junior, but is it even possible to make up for those two years? If so, how can I do it?

          Students doing research together

          You can still turn things around, but you’ll have to work hard. Ask teachers, tutors, and your parents to support you in an effort to raise your grades and turn your education around.

          Your junior year is a crucial year to show schools how much you care about your studies, and how committed you are to your grades and education. Join a study group, get a tutor, or work with your teachers to make sure you’re caught up on your coursework, and try to raise your grades as much as you can between now and graduation.

          Also, don’t discount senior year as a year. Because your grades were lower the first two years, make sure these last two years are meaningful by taking challenging classes n which you can make a “b” or higher, and by studying hard for standardized tests.

          The game isn’t over, but meet with a guidance counselor now and express your goals. There are people at your school willing to help you achieve your dreams if you reach out to them and demonstrate that you are ready to work. Good luck!

          Right now I have a 1.8 unweighted GPA. These past couple of days, I have been worrying, wondering if I will have a successful future. I’m 17 years old and mostly interested in technology.

          Right now I’m taking AP computer science, which is a class on programming with Java, and I do well in the class.

          college students taking an exam

          I’m worried because I’m not doing very well in other areas of school. I know I’m doing poorly in subjects that don’t interest me.

          What can I do? I have 2 years left of high school, and I really want to go to college.

          It sounds like you’ve got a hill to climb, but the good news is that you understand yourself well. You will need to pull up your grades if college is a real goal for you. Can you talk to your teachers to discuss your interests, passions, and goals?

          Perhaps your strengths lie outside the curriculum you are studying, and sometimes teachers can suggest classes and programs better suited to your needs, strengths, and talents. Also, discuss the issue with your parents to see if researching other schools is a possibility. If you are already interested in computer science and programming, there may be a high school with programs geared toward students who excel in these areas.

          It’s not uncommon for students to excel in one area while falling behind in others, especially if they are strongly interested in one subject. But act fast: talk to teachers and parents, and find out what you can do to pull up your grades so that college can be an option for you. Good luck!

          I am wondering if it will be difficult for me to get accepted to colleges. I graduated in 2009 with a GPA of 2.5. I did not realize how important it was to do well in school. After graduation, I went to a community college and have done very well there.

          Two students studying together at home

          I have a 3.6 GPA and have made the Dean’s List every semester. Also, I have an internship under my belt. What will a college think about my low GPA from high school? I also have no ACT or SAT scores. Can I still be accepted?

          The answer is yes. You can certainly still gain admission to a college, but you may need to take the ACT or SAT. That will depend on the college’s admission requirements.

          Editorial Listing ShortCode:

          It is highly commendable that you have raised your grades and participated in one of the summer internships for high school students. A school will likely see your recent transcripts as a sign of improvement and lifestyle change. When determining an admission decision, schools look at a variety of factors including grades, test scores, extracurricular activities, recommendation letters, and essays.

          You can find out about studying for the ACT and SAT online. High scores on the tests your chosen school requires will be an important factor in your application. Study hard, and don’t give up. College is definitely still in reach!

          I did poorly during my freshman and sophomore years at college. I developed epilepsy and my father died. Instead of continuing to my junior year, I decided to go to a different school and restart my schooling.

          I retook all my classes and obtained a bachelor’s degree in science. I am applying to medical school now and my GPA from the last 4 years makes me a competitive candidate.

          Motivated students in a group study

          If I include marks from the first two years, though, I barely make the cutoff. Is there a way around this? There is no way to explain my situation to medical schools if my grades are averaged by a centralized system.

          Once they see my GPA, I won’t be under consideration. Please help or let me know where I can find some help. Thank you.

          I’m sorry to hear about the unfortunate events that occurred during your college career. It sounds like you made some wise choices and were able to complete a degree with high grades. When you apply to medical school, you will have to report transcripts from all of your undergraduate coursework.

          Concealing transcripts could cost you an acceptance you worked hard to earn. Instead, use your admissions essay to discuss the barriers you overcame. This will speak to your diligence as a student and your ability to face challenges and achieve triumphs.

          You will likely have a chance to interview at medical schools as well, so you’ll have ample opportunity to provide a testimony of your achievements to go along with your transcripts. Also, get in touch with an admissions counselor at the medical schools to which you’ve applied and introduced yourself.

          Explain the situation and ask whether you’ll still be considered. It never hurts to make direct contact and build relationships at the admissions office. Good luck!

          I’m a junior in high school and am worried because I’ve done poorly throughout the first semester. In previous years, I had a B average, which resulted in a 3.1-2 GPA. This year, however, I’m receiving Cs and a few D+s.

          Junior high school students reviewing their notes

          The prospect of getting into college has recently clicked for me and I’m having trouble focusing because of the stress to improve on them.

          I think I’m typically a well-rounded student, but I need some guidance because the first semester is pretty much over and I don’t know what to do.

          It sounds like you’re putting forth a lot of effort but not seeing the results you want. It may be time to reassess your schedule and congratulations for recognizing that a change may be in order. Typically you should only take honors or AP courses if you are able to earn a ‘B’ or higher.

          If you find that your grades are dropping as low as ‘Ds’ or even ‘Cs’ you ought to scale back on the number of AP courses you are taking. Before you become overwhelmed and dismayed, make whatever changes you can make to your schedule. Also, call on help from guidance counselors, teachers, and tutors. This is a good time to solicit all the resources and help you need because grades during junior year are fairly crucial in the admissions process.

          Good luck, and don’t give up. It sounds like you are a great student who simply needs a few adjustments to your schedule!

          Will I definitely get a bad job if I receive low grades in high school? This question really bothers me. I’m worried about my future.

          Classmates talking about their lessons

          Not necessarily. It depends on how you apply yourself now. Your career should be something you are passionate about, and when you find your passion, hopefully, you’ll be willing to work toward it. Meanwhile, you could start at a community college and take the starting credits toward a degree.

          Most community colleges accept students with lower grades. From there, research university programs or technical trades that are aligned with your job interests. If you know there is a job out there you’d love to have, do some internet research to find out what kind of education and training is required. Then, aim toward your goal and work hard. It won’t be easy, but it will most definitely be worth it. Good luck!

          My daughter is considering dual enrollment for her junior and senior years of high school. I am wondering if it would be better for her to stay at her high school full time and enroll in several AP classes (she has straight As in her current AP classes).

          Which would be better for her GPA and for her college applications?

          That’s an important decision, and the answer depends on how well you think your daughter will respond to the new challenge of college-level work. Dual enrollment can jumpstart a college career by preparing a student for the rigor of college study, but if it will interfere with her ability to keep high grades in high school, it may not be a good move.

          Editorial Listing ShortCode:

          It sounds like your daughter is making high grades in AP classes. If you determine whether she’s skating through easily with As, or studying hard for those grades, you might be able to measure whether she’s ready for the next level of challenge. I would certainly step into dual enrollment slowly, taking on no more than a few credits at first to see how well she does. Good luck with your decision!

          How can I start over at a community college if I went to a university two years ago and had a 1.0 GPA? I was taking classes that are no longer relevant to what I needed. Do these still matter?

          University students looking at their notes

          You can probably still start over, but you’ll need to apply to the community college and report all past transcripts. It’s crucial that you include these past transcripts in your application, even though they might not reflect the best grades.

          If you have a chance to write an admissions essay, you can explain the low grades and discuss why you are now ready for college-level work. Good luck with your application process!

          I have always worked hard in school and I have maintained a 3.0 GPA throughout high school. I recently took the ACT and made a 26 which could probably get me an academic scholarship, but my GPA is not high enough to qualify. What should I do?

          Strengthen all other areas of your application as much as you can, and use your essay to discuss your grades and why they are not as high as your test scores.

          Perhaps you’ve taken challenging classes or worked during high school? Or, perhaps you have participated in extracurriculars? If you have an element in the application that can’t be changed, it’s best to focus on what you can change at this point. Ask for strong recommendation letters from teachers who know your work and your goals well. Good luck to you!

          I am interested in a semester study abroad program next year (my senior year). Right now, my grades are mostly As and Bs. My GPA is about 3.0.

          Because the possibility of credits not transferring while abroad exists, would it be a mistake to leave my senior year and miss out on raising my GPA, or would a study abroad experience enhance my college applications?

          Students preparing on taking an exam

          Making this decision will require a little critical thinking about the college program you’d like to attend and the career goals you’d like to work toward.

          For example, if you plan to study international studies, foreign languages, political science, or a field where international travel is paramount, then studying abroad might be more important to have on a transcript than a few more As in regular high school classes.

          One thing you could do is contact an admissions representative at your school of choice and gather his or her opinion. While a 3.0 isn’t a low GPA, pulling it up a bit wouldn’t hurt your chances of success. However, you’ll want to weigh the actual amount you’d be able to raise your GPA before applications are due with the enriching experience you might gain from studying abroad.

          Good luck with your choice! Either way, make the most of your experience, whether you stay or go.

          I am a junior. I have received some college invitations for sports. When I go for a visit, will these colleges look at my current grades in progress? I have time to improve before the end of the semester.

          Congratulations on your invitations. The answer is, yes, schools will be interested in your current and past grades. Junior year is an important year, as it marks the point in your high school career when you begin to consider college studies seriously.

          Schools look for an improvement in grades and an established schedule of extracurricular activities by junior year. It’s great that you still have time to improve. Work as hard as you can to keep your grades high, and solicit tutoring if you need extra help. Competition for college is tough, so even talented athletes need strong grades going into the admissions process. Good luck to you! Keep working hard.

          I am currently a junior. During my freshman year, I failed two classes. My sophomore year was not much better, as I skipped a lot of school. But now, as a junior, my grades are better and I’m serious about wanting to go to college.

          Students working together on a project

          I have changed. I really want to enter a four-year university but from what teachers have said in class, I’m worried that I won’t be able to get in.

          If I straighten out these last two years, will I be able to enter a university? I’m losing my mind because of the fear I won’t succeed in life.

          First off, don’t lose your mind! Success in life depends on what you do with the circumstances at hand, so success is never out of reach. That said, aiming for four-year university education will be difficult with a record of low grades.

          It’s not unattainable, but you will have further to climb considering your past records. Don’t let that discourage you: Let it motivate you. Many students begin college in a community college and then transfer to a four-year university when ready. This might be a more suitable goal to set. It’s great that your grades have improved.

          Editorial Listing ShortCode:

          You might consider meeting with a counselor or advisor and discussing your goals. He or she can help you build a better transcript, develop study habits, and add extracurricular activities to the equation. Additionally, high scores on the ACT or SAT will be important to balance out lower grades. If you’re serious about college, go to the resources at your school and find someone who can help you make a plan.

          Your chances are not lost, but get ready to put in some serious work. When you do apply to schools, be prepared to discuss your growth in an essay, and apply to a range of schools, not just the top picks. You may receive some rejections, which is normal, but you’ll widen your chances by considering a variety of schools. Good luck to you!

          I have a feeling that I will not be able to make it into the college I’ve chosen with a 3.5 GPA. I currently have a B- in an AP class.

          To improve my grades and have two extra hours for studies, I may have to drop out of a sport I really love. I’m not sure whether this is a good choice.

          Attentive students listening to their Prof

          First off, don’t get discouraged. You’ve still got a solid GPA even though your grade has declined in the AP course. It’s great that you recognize that you need to make a change to shift your study habits. Typically I would only recommend taking an AP class if you can achieve a B or higher in that class. It sounds like you’re right below that point, but a few extra hours per week of studying may help you raise your grade.

          If you’re great at a sport and want to continue that sport in college, I don’t advise quitting. However, if the sport is something you know you’ll stop once you finish high school, it seems like a sound choice to put more energy into your studies.

          You can explain in your application essay that you gave up the sport to allow yourself more time to study and earn higher grades. But first, take a long hard look at your schedule and see if there are other things can move aside to allow for studying. It sounds like you really love this sport and are talented.

          Perhaps you’re spending time on Facebook, on the phone, or doing other activities that don’t serve your future, and you could find extra hours to study by reassessing your day. I know it’s a hard decision, so make sure it’s the sport that is cutting into study hours before you drop it. Good luck!

          I’m in a bit of a rut. I’m a junior in high school and my grades are low. I feel that it’s too late for me to get accepted at my desired college.

          However, if I attend a college for one year and then transfer to my desired university, will the university look at my current college grades or will they look at my high school GPA? Or, do they use both grades to determine my place there?

          Students walking together after class

          I’m sorry to hear of your dilemma. The answer is this: They’ll look at both. However, if you show great improvement from your last year in high school to community college, it is likely that a university will interpret that improvement as maturity and growth.

          Many students find a way to become more organized and ambitious later in high school, and college committees know that. The best route for you to take now is one of earnest improvement. Get a tutor if possible, and try to raise those grades. Also, as long as it doesn’t interfere with your studies, think about joining a few clubs or organizations that interest you.

          You’ll meet other students and you may find it easier to excel in class with a better sense of community. In sum, don’t give up yet! You still have time to show that you’re serious about your education and goals. All it takes is a little effort, focus, and belief in yourself!

          I am finally finished with my junior year, and my overall GPA is about 2.7. I am 99% sure that I’ve blown all my chances to get into college. I play basketball at a very high-quality school, and to get a scholarship at a great school would be my dream.

          I take the SAT and ACT soon, and I’m hoping if I study really hard that I will get a high score. What should I be thinking right now?

          Student showing notes to her classmates

          You should be thinking positively and working hard! Of course, you have not blown your chances, but you do have a steep hill to climb to overcome a low GPA.

          Do what you can between now and graduation to raise your grades, and study hard for the SAT and ACT tests. If a basketball scholarship is realistic for you, put some energy into researching schools and meeting with your coaches to learn what opportunities are out there. The most important thing is this: Don’t give up!

          I am a freshman in high school and I am currently taking all honors courses with all A and A- grades. I take two dance classes a week, I just finished a musical in which I had a small singing/speaking role.

          students walking together in the hallway

          I would like to either volunteer at a hospital or get a job this coming summer. However, right now I am questioning whether I should take part in the state drama competition through my school this winter.

          I know that I would fully enjoy myself throughout this activity, but because of the commitment, my grades would probably drop slightly.

          Is it worth it to skip this drama competition in order to keep my grades up even though I won’t be enjoying myself very much?

          I would love to get into a top school and eventually become a pediatrician, but I am not sure if it is worth it to risk my enjoyment for the next few months in order to get slightly better grades. Thanks!

          Tough decision! But, it’s important to know that schools look at extracurricular activities just like they look at grades and test scores. Doing something you’re passionate about and setting a goal outside of academics speaks loudly about your character, in a good way!

          Editorial Listing ShortCode:

          If you feel like your grades will suffer tremendously, of course, you should stick with academics. But, is there a way you can adjust your time management so that this competition doesn’t interfere with your grades? I don’t see why it has to bring your GPA down, and if you’re really passionate about the drama competition, you can perhaps readjust other parts of your schedule to accommodate your studying.

          The choice is yours, but rather than seeing it as a binary with two definite choices, think about what you can do to keep your grades up and follow your passion! Good luck!

          I come from a decent high school, ranked highly in national rankings. I messed up in one year and got a D in an honors class. I’m currently retaking this exact class and doing well. I was wondering how private schools consider “retakes” in their calculation.

          Teacher and students in a discussion

          My overall GPA is a 3.7 but I did get a C along the way. Despite this, most students in my year struggled and I ended up in the top 20 students in my class.

          How do colleges view your grades in relation to the context of your school and class rank?

          If you attend a decent school, obtained a high rank, and got some bad grades, would your academic strengths be doubted, or would they understand?

          The best answer is this: Work as hard as you can to raise your grades. It is great to retake the class, and that will not only help your GPA but it will also give you the knowledge and skills you missed the first time around. Trying to predict how colleges will “look” at things is pure speculation and won’t help you reach your goal. Sure, the rigor of a curriculum will be taken into account when you apply to college.

          Let that inspire you to work hard to raise your grades. Once you get to college, success will be more about how you apply yourself than about what your transcripts look like. In other words, you won’t be competing for the best GPA, you’ll be absorbing and learning as much as you can in order to face the working world.

          When you apply, you will have a chance to explain your dip in grades in your application essay. Think about ways to discuss the rigor of your school in a positive way, rather than “blaming” your grades on the school’s difficulty.

          You might discuss how a harder curriculum prepared you for college and gave you the study habits you’d need, and retaking a course allowed you to understand your mistakes the first time around. Good luck to you!

          Students working together on a research project

          I received a C in my AP literature and composition class for our first grading quarter. I am having an extremely difficult time grasping the material.

          I am a straight-A student and have earned As in my other AP courses. I’m considering switching to an honors English class to avoid dropping my GPA.

          Would it be wiser to switch classes or stick it out in AP English?

          I’m sorry to hear about your struggle! We typically advise students to take an AP course only if they can earn a B or higher in the class. Dropping below a B could hurt your GPA, and it shows that you might not be ready for the rigor of that particular class.

          It sounds like you’re a strong student in other fields, so perhaps changing to honors English and pursuing AP classes in your stronger subjects is the best idea. Good luck with your decision!

          Hi! I’ve applied to colleges and my average is about C or a little higher right now. Do I still have a good chance of getting into college?

          Students in a group study

          While I can’t tell you your chances, I can tell you not to give up. Grades are not the only factor in college admissions, and different schools have different GPA requirements for entering students.

          If you’ve applied to schools that accept students in your GPA range, you should feel confident about your applications. Only you can know whether you truly put your best work into the application essay, your test preparation, and your coursework in high school.

          Now, it’s time to focus on the future, consider how you’ll use the next few years of study to boost yourself into an exciting career and keep improving on those areas where you’re not so strong. If you don’t get in, don’t get discouraged.

          Look at local community colleges and see if you can start there to earn coursework to improve your grades. If a college degree is your goal, you’ll get there if you put in the hard work and consistent motivation. Good luck!

          What looks better to colleges: AP classes with lower grades or regular courses with high grades? I am taking AP classes and am passing but with Cs and Bs. If I took the regular class, I would be guaranteed an A.

          Do colleges prefer high grades in regular classes or average grades in advanced classes? Is your GPA the most important thing they look at?

          Three males students studying for AP class

          This is a great question that many future applicants have. First, AP and honors classes are designed to challenge students who are ready for a higher level of learning.

          Unless you can earn a B or higher in these courses, it is not generally advised that you take on the challenge. They are more difficult and demanding than regular classes but are also better prepared for college coursework. A college will factor in the academic rigor of your transcript when assessing your grades, so a B or higher in an AP chemistry class might hold more weight than an A in a regular-level class.

          Editorial Listing ShortCode:

          Remember, though, that the GPA is only one of many factors the school will consider including test scores, recommendations, essays, and activities outside of academics.

          I didn’t do very well in my freshman year. I didn’t fail any classes, but my grades were not the best. I had mostly Bs with a few Cs and a D. Throughout the rest of high school, I earned all As and Bs.

          Is it still possible for me to earn a 3.5 GPA? Will I be able to go to a good college? I want to become a nurse, but I also want to get into a good college.

          Every school calculates GPAs differently, so if you want to know what GPA is possible for you, you’ll need to talk to your advisor at your school. However, I can tell you not to give up your dream of becoming a nurse.

          Many nursing programs are highly competitive, but you can aim for ones that accept students within your GPA range. Just make sure the rest of your application is full of stand-out qualifications, such as strong recommendation letters, high test scores, and a well-thought essay. Good luck, and don’t give up on your dream!

          I am a junior at a magnet school and I’m also in 3 AP courses. AP Biology has always been my biggest struggle, and my grade at the end of the 3rd semester might be a D. How can this impact how colleges consider accepting me?

          Students taking AP classes

          While the junior year is a crucial year to earn high grades and round out your profile as a college hopeful, it’s not the end of the world to struggle in an AP class. If you can’t maintain a ‘B’ or higher in an AP class, it is usually wise to revise your schedule and take a less advanced course instead.

          Knowing that you can be warier of AP classes in the coming semester and avoid taking on too much. Do everything you can, now, to raise that grade and seek help from teachers and tutors. When it’s time to apply for colleges, hopefully, you’ll be able to demonstrate how you overcame those challenges and raised your grades. This is something you can discuss in your application essay. Good luck and don’t give up!

          I started my college career at a 4-year university but I did horribly because I was immature and didn’t take things seriously. I was on academic suspension so I transferred to my local community college and I’m currently doing very well.

          Students talking at school campus

          I want to transfer to a different 4-year university now that I have grown up and learned to take college seriously. But I’m afraid that I won’t get accepted anywhere because of my abysmal GPA at my first school.

          What should I do to get accepted?

          Congratulations on turning things around! From what you’ve written, it sounds like you’re very honest about what went wrong the first time. My best advice is to write a strong admissions essay that discusses the ways in which you’ve changed.

          If you can demonstrate that your most recent academic record reflects the “real you” who understands the importance of your goals and education, you’re more likely to win a spot at the four-year college you want to attend. Also, try to solicit recommendation letters from professors who know how serious you are about your education. These are likely professors who know your future career goals. Good luck!

          I’m nearing the end of my freshman year and I have all As except for drama. Drama is supposed to be an easy A, but the teacher hates me for some reason and yells at me constantly.

          I will probably end up getting a B in the class and it will affect my GPA. When colleges look at my GPA will they see that my only B was in drama and not care?

          Students sitting on bench and doing research

          It’s not that they won’t “care,” but it won’t likely make or break your acceptance to a school, especially since you’re only a freshman. But, more importantly, can you meet with the teacher and try to find out how you can do better in the class? I understand that you feel like the teacher hates you, but this just isn’t likely.

          It’s not common for a teacher to have a personal vendetta against a student, and perhaps there is a misunderstanding or a way that you could participate more effectively to make the grade you want to earn. Truly consider meeting with the teacher to find out how you stand in the class and be honest and open about your desire to do well.

          Editorial Listing ShortCode:

          You might get great results. Also, try to avoid calling a class an “easy A.” No class is an easy A, whether it’s drama or organic chemistry. You’ve still got to put in the work and engage in the material. Odds are, your drama teacher did not study theater and literature in order to teach an “easy A” class. He or she is likely passionate about the subject and quite wary of students who fail to take the class seriously.

          Put yourself in the teacher’s shoes: Would you want someone to treat your career and profession like an “easy A” filler class? Good luck talking to the teacher!

          I’m at the beginning of my senior year of high school and I’m looking into different colleges. My problem is my GPA: it’s looking towards a 2.3 weighted at the end of this school year.

          Group of students discussing about their project

          In most of my classes freshmen and sophomore year I received low grades in general studies, but as an upperclassman, I’ve gotten on a good track with mainly honors and AP courses.

          I have an opportunity of retaking many of my previous classes online through the school to at least get a 3.0. Should I take it or should I show my upward progress as suggested to me by one of my advisors?

          First off, great job increasing your grades! It can be hard to get ahead once you’re behind so it’s clear you put a lot of effort in and that should be commended. It’s true that schools do love to see upward progress and it’s terrific you’ve shown that. However, retaking those classes to boost your GPA sounds like the right way to go.

          Yes, a school may be impressed with your trajectory, but many look at the overall GPA first so you run the risk of not being seriously considered based on that, and they may never get to poking into the particulars of your journey and seeing how well you applied yourself going forward. It’s a big gamble to rely on them making those inferences.

          Taking the classes again will show the tenacity colleges look for when they offer spots to students they believe will be able to succeed. But there is still a way to parlay that upward climb into winning points with schools. It will likely make excellent fodder for an essay as you discuss how you were able to raise your GPA through extra effort,  and how you realized the importance of starting strong to finish strong.

          I wish you much luck in your senior year,  and the spirit to retake and excel in those classes!

          Hey Guru! I just began my junior year in IB, and so far my grades are not that great. It’s still only half term, but I used to have amazing grades and this year I think I’m just not studying that much.

          Professor leading the class meeting

          In addition to harder courses, I also have a boyfriend now. I’ve had 3 tests so far: chemistry, math, and econ, and failed all of them.

          Even though we are still going to have a couple of other tests this term in each class, I’m worried I’ve already ruined all my chances of getting good grades and going to a good university.

          What do you think?

          Junior year is an important year for high schoolers, and you’ve already taken an important step by taking International Baccalaureate (IB) coursework. Colleges are interested in good grades, sure, but they also want to see that students continue to challenge themselves with rigorous classes instead of just taking the easy road.

          At this point in your junior year, you need to decide if you’re capable of stepping up and focusing on your studies. You’ve gotten good grades in the past, so I sense that you can do it if you put your mind to it. If college is your priority, then it’s time to reorganize your life so you give your studies adequate attention.

          Schedule time for your new boyfriend on the weekend, and focus on bringing up your grades this term. Most colleges are understanding a rough patch in a transcript. In fact, a short period of lower grades followed by a steady increase can demonstrate resilience and perseverance. Don’t give up now! You are capable of pulling yourself out of this hole and finishing your junior year strong. Good luck!

          I am a junior at one of the largest schools in the United States, with around 4000 students. Class rank is not specified, but I am in approximately the top 3.5% of my class at least. My weighted GPA is 4.63 and my unweighted GPA is 3.92.

          I will finish with 11 AP courses. Right now, I am taking AP biology and am doing terribly. I am suffering from a low ‘B’ and am fearing the possibility of getting a ‘C’.

          Friends arranged a group study for biology class

          Will this one blotch in my transcript hurt my chances significantly of getting into a top-tier university?

          I know that there are so many factors that contribute to college admissions, so will one ‘C’ really hurt my chances of being accepted into an Ivy League School for example?

          While I can’t predict your chances of getting into any school, I can tell you that no single grade will make or break your chance of being admitted to college. It sounds like you are a bright student with a rigorous schedule. That’s great!

          AP classes look great on a transcript, and a committee will likely see you as a student who is hungry for challenge and excellence. What’s more, if this one grade is truly different than your track record as a whole, it will be easy to see that you struggled with this one class, rather than experiencing a decline all-around in your academic performance.

          Editorial Listing ShortCode:

          Do everything you can to raise the grade, and seek help from a tutor or from your teacher. But don’t panic: If you give it your best work, you can walk away knowing that you did everything you could. That’s the practice that will make you the most successful in college and in your career.

          In the future, if you feel you cannot make a ‘B’ or higher in an AP class, it’s advisable to adjust your schedule accordingly. Keep working hard and balancing your extracurricular activities with solid grades, and you will likely have great results when it’s time to apply to schools! Good luck!

          I am currently a junior in high school. My high school is ranked #1 in the state of Kansas, I’m taking two IB courses and plan to take four next year. But to stay at my school which is a magnet school you have to keep a 2.5 GPA.

          I always had a 3.8 GPA at my middle school but in my freshman and sophomore years, I’ve been struggling to make the required 2.5.

          Group of students brainstorming for their project

          I’m now working hard and making a 3.0 unweighted GPA. I plan on applying to the University of Missouri, Kansas State University, and others along this line for computer science.

          Are colleges going to take my highly ranked school into account or should I just leave school and go to a nearby school, which was ranked one of the worst in the state?

          It sounds like you are working very hard. Yes, admissions committees have data on the schools from which you apply, and they do take into account the rigor of the school you attend.

          Keep working hard, and know that working hard for a 3.0 is much more valuable than “skating by” to earn a 3.8 in easier courses. Your determination, drive, and the value you place on a college education will motivate a lot of your success in the admissions process, and those are things you can show in an essay or an interview (if the school requests one.)

          I would not advise you to leave the school and attend one that you consider to be the “worst in the state.” That would be a poor choice if a solid education is what you seek. Stick with your school, and get some help from other resources, parents, tutors, or teachers, to continue raising your grades and studying hard. Good luck!

          I am currently a junior in high school. In my freshman and sophomore years, I had a stellar GPA and took all of the hardest classes my school offered.

          This year, though, my GPA is less than ideal due to many circumstances outside of my control (teacher dying, my own illness, family problems, etc.).

          Students studying for the test

          However, it is important to mention that I am taking 4 college-level classes. My ACT score and extracurriculars are pretty good.

          I am concerned that my junior year GPA will ruin my chances of getting into a selective college. I am extremely stressed out about this so any help would be much appreciated!

          Thanks for reaching out. It sounds like you’ve been through a lot! I can also see that you’re a committed student, and I suspect that your GPA in your junior year will be better than you think.

          Keep in mind that colleges will look at the fact that you’re taking college-level classes and that will be taken into account in evaluating your GPA. So, for example, a “B” in one of those courses may be viewed as an “A” in regular courses. Also, on your college application, you’ll be able to explain the circumstances that may have contributed to your GPA being a little lower during your junior year if that even turns out to be the case.

          Don’t stress out! You have a strong GPA, test score, and extracurriculars, so I wouldn’t worry about your junior year GPA hurting your chances of getting into a really good college.

          Good luck!

          I am a rising junior year in high school. I really struggled with one of my honors classes, and I ended up getting a D in it both semesters. Should I get out of the honors class and go to the regular class?

          Also, is there a chance for me to get into a good college if I am able to pick up my grades in my junior year?

          Professor and students discussing in the library

          Colleges are impressed by honors courses, but not every class you take needs to be honors. It’s more important to take the most challenging course that you can actually succeed in.

          Although it’s typically better to work at keeping up your grades in an honors course, in this case, you might want to consider dropping down to the regular course next year. Generally, I wouldn’t take an honors class unless you’re confident that you’ll earn a B or higher. And of course, you can still get into a good college! One low grade is not going to hurt your chances of getting into a four-year college.

          Editorial Listing ShortCode:

          Plus, colleges look at more than your GPA when considering your application. Fortunately, you have two more years to bring your grades back up. Although every semester matters, the college will be impressed if they see your grades increasing. Focus on improvement. Join a study group or get a tutor to try to raise your grades as much as you can, and make sure to balance your studies with extracurricular activities that you’re passionate about.

          College is definitely still within reach. Good luck!

          I am about to enter my junior year of high school. My freshman year grades were great, and I had an unweighted GPA of 3.6.

          Students discussing their research

          My sophomore year, however, took a turn for the worst. I received my first D, in Algebra 2, and it doesn’t stop there. Throughout the year, I received Cs in chemistry and Advanced Placement world history as well (all of my classes were honors or AP).

          In my sophomore year, my highest GPA was a weighted 3.1, and my lowest was an unweighted 2.5. I worked as hard as I possibly could.  I’ve just never taken such demanding, difficult classes.

          I plan on taking easier classes in my junior year. I’m only taking two AP classes and honors English and anatomy.  If I maintain above a 3.5 in my next two years, will colleges turn me away?

          I do a lot outside of school. I’m in a lot of clubs and extracurricular activities, and I also play volleyball. I just want to know if my chances of getting into a good college are completely ruined by my sophomore year.

          Your grades can make or break whether you get into a top college, but don’t panic! You still have time left to raise your grades. Have you considered working with a tutor or asking teachers for extra assistance outside of class? Colleges take into consideration your entire high school career.

          Even though you struggled through the second year of high school, if colleges see that you worked hard and improved your GPA, they will be impressed. That being said, you said you’re planning on taking easier classes next year, but you’re still signed up for higher-level courses.

          Although colleges are impressed by the rigor of classes you take during high school, I wouldn’t recommend taking those AP and honors courses unless you’re confident that you’ll earn at least an A or a B in them. Also, colleges look at more than just your grades.

          Of course, your test scores will be an important factor, Also,  if your extracurricular activities demonstrate your commitment, maturity, and leadership, that will make a big impression on college admissions committees. Your personal essay and recommendations will also be considered.

          If you work hard to improve your GPA in the next two years and score well on the ACT or SAT, there are many schools out there that would be lucky to have you. Stay focused on improvement.

          Good luck!

          I’m an incoming sophomore, and I go to a school in New York. I failed my geometry regents, and I also failed the class because I did really badly on the final.

          I asked my guidance counselor if colleges would see that I had to take summer school, and she said they would see it as a summer credit.

          Is that just a nicer way of saying they will see that I had to go to summer school to retake it? I really don’t want a failing grade to restrain me from going to the college that I want to go to.

          Students reviewing notes for geometry class

          I’m sorry to hear that you struggled in geometry class, but one bad grade isn’t going to keep you from getting into a good college or university.

          If you are able to go back and retake the course in summer school, go ahead and do that. Although colleges will see that you attended summer school, if you pull up your grade in the course, they will also see that you were able to get serious about your studies and overcome your challenges.

          Colleges like to see students with high grades as well as students who show a steady track of academic improvement. You have three more years to show that you are that student. If you want to increase your chances of getting into the college that you want to go to, I would suggest strengthening all of your grades, nailing your test scores, and building relationships with teachers who will help motivate you to succeed.

          Also, colleges also look at more than just grades. To increase your chances of getting into your first choice of schools, consider volunteering at a community organization or taking on a leadership role in a school club. A well-rounded extracurricular history will boost your college application.

          Freshman year is a learning curve. Stay focused and positive. If you want a college education, you can work toward that goal and earn it. Good luck!

          My school is a Jewish school that makes us take three foreign languages: English, Hebrew, and Spanish. My grades are fantastic (As or above in almost all subjects); however, in Spanish, I got a B- freshman year and a B sophomore year.

          Students doing research in the school library

          My school lets us drop Spanish after sophomore year and instead take economics. Because I am interested in math and going to a business school, I’m wondering whether to drop Spanish because I already have a foreign language in Hebrew.

          Foreign language requirements vary from school to school, so whether only two years of Spanish will be a disadvantage will depend largely on where you apply.

          Most selective colleges and universities require at least two years but recommend four years of the same language. Oftentimes, it’s not so much about the grade. Learning a foreign language allows you to engage with another culture and understand its ideas and values. Sure, you learn vocabulary and syntax, but you also read novels and poems, and magazines, preparing you for our increasingly globalized world.

          Editorial Listing ShortCode:

          If you really despise your Spanish class, you might consider taking economics instead, especially because you’re also learning Hebrew. However, a B- isn’t a terrible grade, and if you continue to improve throughout your final two years in high school, you’ll send the message to college admissions counselors that you’re up for a challenge and interested in having a broader understanding of other cultures.

          A demonstrated proficiency in a second (or third) language will only strengthen your application. If you’re not aiming for the most selective schools, and if your application is strong in all other areas, you can get away with just two years of Spanish.

          For a list of the language recommendations and requirements for some of the schools that you might be considering, check out the College Board website.

          I’m a sophomore in high school. When I was filling out my high school course request I signed up for all of the advanced courses because I got good grades in advanced classes in my freshman year.

          Students taking advance classes

          This year, my advanced classes are a little bit more challenging. The first semester is about to end and my grades aren’t what I’d like them to be.

          I’m panicking because I want to get into a good college and I’m worried my grades are going to keep me from getting into a good school. Please help!

          The transition from middle school to high school can be tough. As you continue to progress to higher-level courses in high school and college, some of your classes are going to get more challenging.

          The good news is colleges know that advanced classes are more rigorous than regular placement classes in high school and take that into consideration when reviewing college applications. Colleges would prefer to see that you are challenging yourself with advanced courses and see that you are doing well academically rather than see perfect grades in all regular placement classes.

          Imagine a college looking at the transcripts of two different students: one has a 4.0 in regular classes and the other has a 3.7 in Advanced Placement classes. Even though the student in regular classes has perfect grades, the student with a 3.7 GPA might be seen as more competitive because they are taking more challenging courses.

          That being said, if you feel like there are some subjects you just aren’t able to thrive in advanced classes, you may want to consider taking regular classes in those subjects. In general, it’s not advisable to take an AP course unless you’re confident you can get at least a “B” in it.

          Education isn’t one size fits all. Maybe you are stronger in some subjects and weaker in others. Keep in mind what you need to learn and succeed. If you’re worried about the grades you’ll end up with this school year, there is no shame in getting a tutor!

          I am a junior in high school right now and I am really nervous about college applications. My grades are average this year but my grades were terrible last year. This year I have started treatment for depression, anxiety, and ADHD.

          Students clarifying something with their teacher

          It has made a huge difference, but I am still just average (my GPA is 85). I am so worried about getting into a good college because I’ve been so dedicated to my studies and I fear it won’t pay off. How do I stop worrying about college? What are my chances of success?

          Congratulations on improving your grades! If you’re using the typical grading scale, your 85% GPA would be equivalent to a 3.0 or “B” which is above average and considered to be a competitive GPA by many colleges and universities.

          You’re doing well and still have time to improve your grades before you apply to colleges. Many top colleges require a 3.5 to 4.0 GPA to be considered, but many good schools that are a little bit less selective would find a 3.0 GPA to be accepted for admissions.

          College admissions are difficult and getting accepted into a school isn’t ever guaranteed, but it helps to not get caught up on one particular school and come up with a list of several schools you would be happy to get into in order to reduce the application anxiety, including some “safety” schools.

          I’m currently a junior in high school with a 3.38 GPA. I’m worried I’m not going to get accepted into colleges because I didn’t perform well in my freshman and sophomore Honors and AP classes.

          I received several Bs and Cs as final grades. However, I have done very well on standardized tests and I participate in 3 extracurricular activities.

          Students talking as they enter school campus

          My junior year has been stellar thus far with 2 As in my AP classes and Bs in my honors classes. What are my chances of getting into college? Is there anything I can do to decrease my chances of getting denied? 

          A 3.38 unweighted GPA is a solid GPA for a lot of colleges! It is important to note, however, that there is a difference between a 3.38 unweighted or weighted GPA. If your GPA is a 3.38 weighted, you may want to take a look at your unweighted GPA to get a better idea of which schools require a GPA in your range. 

          Assuming your GPA is unweighted, you are above the national average and should be able to get into a variety of good schools. Although, many “top” schools may require higher GPAs. As far as increasing your chances of getting accepted, striving to do well in school and be a well-rounded student is important. It seems like you’re already trying to do both of these things!

          Editorial Listing ShortCode:

          Something else you may want to keep in mind is that colleges do know that Honors and AP classes are more difficult than regular courses in high school. A “C” in an AP class is still better than a “C” in a regular class.

          It is recommended to only take courses at an Honors or AP level if you are confident that you will be able to pass them with an “A” or “B”. Applying this knowledge, you may want to select classes you’re confident about doing well in when you choose your senior courses.

          If you are still worried your GPA is too low to get into the programs you’re looking for, you may want to consider going to a community college for two years to improve your grades and then transfer. This could be a good option if you don’t mind starting out at a community college in order to be able to transfer to another four-year college or university.

          I’m currently a junior in high school with a 3.38 GPA. I’m worried I’m not going to get accepted into colleges because I didn’t perform well in my freshman and sophomore Honors and AP classes. I received several Bs and Cs as final grades.

          However, I have done very well on standardized tests and I participate in 3 extracurricular activities.

          Classmates working on the activity together

          My junior year has been stellar thus far with 2 As in my AP classes and Bs in my honors classes. What are my chances of getting into college? Is there anything I can do to decrease my chances of getting denied?

           A 3.38 unweighted GPA is a solid GPA for a lot of colleges! It is important to note, however, that there is a difference between a 3.38 unweighted or weighted GPA. If your GPA is a 3.38 weighted, you may want to take a look at your unweighted GPA to get a better idea of which schools require a GPA in your range. 

          Assuming your GPA is unweighted, you are above the national average and should be able to get into a variety of good schools. Although, many “top” schools may require higher GPAs. As far as increasing your chances of getting accepted, striving to do well in school and be a well-rounded student is important. It seems like you’re already trying to do both of these things.

          Something else you may want to keep in mind is that colleges do know that Honors and AP classes are more difficult than regular courses in high school. A “C” in an AP class is still better than a “C” in a regular class. It is recommended to only take courses at an Honors or AP level if you are confident that you will be able to pass them with an “A” or “B”.

          Applying this knowledge, you may want to select classes you’re confident about doing well in when you choose your senior courses. If you are still worried your GPA is too low to get into the programs you’re looking for, you may want to consider going to a community college for two years to improve your grades and then transfer.

          This could be a good option if you don’t mind starting out at a community college in order to be able to transfer to another four-year college or university.

          Ready to start your journey?
          Elizabeth Abner
          WRITTEN BY Elizabeth Abner

          Elizabeth is pursuing a Doctor of Philosophy in Foreign Policy and earned her master's degree in business administration. For her undergraduate studies, she earned a Bachelor of Arts in Business Administration with a concentration in international business. Elizabeth's research is focused on universities offering online degree programs.